Sunteți pe pagina 1din 10

lgebralineal

Ejercicio 2
Determina en cada caso si el subconjunto dado H del espacio vectorial V es
un subespacio vectorial de V.
1. V = R2: H = {(x, y) tales que x = y}.
2. V = R2: H = {(x, y) tales que x2 + y2 1}.
3. V = Mnn: H = {A Mnn donde A es triangular superior}.
a b

4. V = M22: H = A M 22 tal que A =


.

b c

3.4. Combinacin lineal y vectores generadores


de un espacio vectorial
En esta seccin veremos cundo un conjunto de vectores puede generar
un espacio vectorial. Para esto necesitaremos los conceptos de combinacin
lineal, conjunto que genera y espacio generado.

Definicin 3.3. Sean v1, v2, ... ,vn vectores en un espacio vectorial V.
Entonces cualquier vector de la forma v=a1v 1 + a 2v 2 + ... + a nvn
donde a1, a2, ..., an son escalares, se llama combinacin lineal de v1, v2, ..., vn.

Ejemplo 3
a) Consideremos los siguientes vectores en R2, (1, 0) y (0, 1), entonces
cualquier vector de R2 se puede escribir como combinacin lineal de (1, 0) y (0, 1)
ya que
(x, y) = x (1, 0) + y (0, 1).
b) En R3, (7, 7, 7) es una combinacin lineal de (1, 2, 4) y (5, 3, 1) ya
que (7, 7, 7) = 2(1, 2, 4) (5, 3, 1).

113

Unidad 3
3 2 8 1 0
c) Consideremos en M23
= 3
1 9 3 1 1
3 2 8
1
lo que
es una combinacin lineal de
1 9 3
1

4 0 1
+ 2
5 2 3
0 4 0
y
1 5 2

2
por
6
1 2
.
3 6

d) Cualquier polinomio de Pn (polinomios de grado menor o igual a n) se


puede escribir como combinacin lineal de los polinomios: 1, x, x2 , x3, ... xn1, xn.
Definicin 3.4. Un conjunto de vectores {v1, v2, ..., vn} de V generan a V si
todo vector de V se puede escribir como combinacin lineal de ellos. Es decir, si
para todo v en V existen a1, a2, ..., an, escalares de modo que v = a1v1 + a2v2
+ ... + anvn.

Ejemplo 4
a) En el ejemplo 3a) de la definicin 3.3. vimos que cualquier vector de R2
poda escribirse como combinacin lineal de los vectores i = (1, 0) y j = (0, 1)
de R2 ; por lo tanto podemos decir que {i, j} generan a R2.
b) De igual manera podra probarse que los vectores de R3:
i = (1, 0, 0), j = (0, 1, 0) y k = (0, 0, 1) generan a todo R3.
a b
c) Consideremos
en M22, entonces:
c
d

a b 1 0 0 1 0 0 0 0

=a
+b
+c
+d
por lo que podemos
c d 0 0 0 0 1 0 0 1
1 0 0 1 0 0 0 0
decir que las matrices
y
,
,
generan a M22.
0 0 0 0 1 0 0 1
d) Los polinomios 1, x, x2 , x3, ... xn1, xn
inciso d).

generan a Pn (vase ejemplo 3

e) El conjunto de vectores de R2 H = {(1, 1), (3, 3)} no puede generar a R2.

114

lgebralineal
Considera el vector (1, 0) de R2, si H generara a R2, entonces existiran a y b
escalares de modo que (1, 0) = a(1, 1) + b(3, 3) de donde tenemos el siguiente
sistema de ecuaciones:
a 3b = 1 y a 3b = 0 pero el sistema no tiene solucin, por lo tanto
H no genera a R2.
f) Consideremos el conjunto H = {(2, 3), (1, 2)}. Vamos a ver si H genera a R2.
Sea (x, y) en R2, si H generara a R2, existiran a y b de modo que (x, y) =
a(2, 3) + b(1, 2), de donde obtenemos el sistema de ecuaciones:
2a + b = x, 3a 2b = y
resolviendo el sistema obtenemos que a =
podemos asegurar que H s genera a R2.

2x + y
3x 2 y
, b=
, de donde
7
7

De acuerdo con los ejemplos anteriores, no podemos suponer que cualquier


conjunto de vectores genera a todo el espacio vectorial. La siguiente definicin
nos aclara este asunto:
Definicin 3.5. Sean {v1, v2, ..., vk} k vectores de un espacio vectorial V.
Denotado por gen {v1, v2, ..., vk}, el espacio generado por {v1, v2, ..., vk} es el
conjunto de todas las combinaciones lineales de v1, v2, ..., vk, es decir,
gen {v1, v2, ..., vk} = {v V tales que v = a1v1 + a2v2 + ... + akvk}
Aqu surge una pregunta: el espacio generado por un conjunto de vectores
es un espacio vectorial? El siguiente teorema contesta esa pregunta.
Teorema 3.4. Si v1, v2, ..., vk son k vectores de un espacio vectorial V,
entonces
gen {v1, v2, ..., vk} es un subespacio vectorial de V.

Ejemplo 5
a) Sean v1 = (2, 1, 4) y v2 = (4, 1, 6) elementos de R3 .
Sea H = gen {v1, v2} = {a1v1 + a2v2} se tiene que si (x, y, z) est en H, entonces

115

Unidad 3
(x, y, z) = a1v1 + a2v2 = a1(2, 1, 4) + a2(4, 1, 6) = (2a1 + 4a2, a1 + a2, 4a1 + 6a2)
de donde obtenemos x = 2a1 + 4a2, y = a1 + a2 , z = 4a1 + 6a2 para algunas
a1 y a2.
Usaremos el teorema 3.2 para probar que H es un subespacio vectorial de R3.
i) Sean x = (x1, y1, z1) y y = (x2, y2, z2) elementos de H, entonces existen
a1, a2, b1 y b2 tales que x1 = 2a1 + 4a2 , y1 = a1 + a2, z1 = 4a1 + 6a2 y x2 = 2b1
+ 4b2, y2 = b1 + b2, z2 = 4b1 + 6b2
entonces x + y = (x1 + x2, y1 + y2, z1 + z2) de donde,
x1 + x2 = 2(a1 + b1) + 4( a2 + b2)
y1 + y2 = (a1 + b1) + (a2 + b2)
z1 + z2 = 4(a1 + b1) + 6(a2 + b2)
por lo cual x + y est en H.
ii) Sea un escalar, entonces x = (x1, y1, z1) = (x1, y1, z1) de donde

x1 = (2a1 + 4a2 ) = 2a1 + 4a2


y1 = (a1 + a2 ) = a1 + a2
z1 = (4a1 + 6a2) = 4a1 + 6a2
por lo cual x est en H.
Por lo tanto, H es un subespacio vectorial de R3.
El siguiente teorema nos indica que si agregamos un vector a un conjunto
generador, el conjunto que resulta tambin es generador del mismo espacio
vectorial.
Teorema 3.5. Sean {v1, v2, ..., vn, vn+1} vectores de un espacio vectorial V.
Si {v1, v2, ..., vn} genera a V, entonces {v1, v2, ..., vn, vn+1} tambin genera a V.

Ejemplo 6
Sean v1=(1,0) y v2=(0,2) elementos de R2.

116

lgebralineal
Propongamos que sea F el espacio vectorial generado por v1 y v2 de tal
manera que:
F= gen {v1, v2} = { v1 + v2} y sean y dos escalares, de tal manera que:
(x1 , x2 ) = (1, 0 ) + (0, 2 ) = ( + 0, 0 + 2 ) = ( , 2 )
por lo que x1= y x2=2 que pertenecen a R2 , entoces v1 y v2 generan a F.
Sea v3 = (3,4) tendremos que
(x1 ,x2 ) = (1, 0 ) + (0, 2 ) + (3, 4 ) = ( + 0 + 3 , 0 + 2 + 4 ) = ( + 3 , 2 + 4 )

de donde x1= +3 y x2=2+4 que son elementos de R2 por lo que v1, v2


y v3 son vectores que generan a F.

Ejercicio 3
1. Responde si son falsas o verdaderas las siguientes afirmaciones:
a) (3, 5) est en el espacio generado por {(1, 1), (2, 4)}.
b) (1, 2, 3) est en el espacio generado por {(2, 0, 4), (1, 0, 3)}.
c) Si {(1, 2), (2, 3)} genera a R2, entonces {(1, 2), (2, 3), (2, 3)} tambin
genera a R2.
2. Determina si los siguientes conjuntos de vectores generan el espacio
vectorial dado:
a) En R2: H = {(1, 2), (3, 4)}.
b) En R2: K = {(1, 1), (2, 2), (5, 5)}.
c) En R3: M = {(1, 1, 2), (1, 1, 2), (0, 0, 1)}.
1 0 1 2 4 1 2 5
d) En M22:
,
,
.
,
0 6 0
1 0 0 0 3

117

Unidad 3

3.5. Vectores linealmente dependientes e


independientes
En la seccin anterior vimos cmo un conjunto de vectores poda o no
generar a todo un espacio vectorial. En esta seccin veremos qu condiciones
debe cumplir un conjunto de vectores para asegurar que genere un espacio
vectorial; para ello necesitaremos introducir los conceptos de conjunto
linealmente independiente y dependiente.
Definicin 3.6. Sean {v1, v2, ..., vn} n vectores de un espacio vectorial V.
Entonces se dice que los vectores son linealmente independientes si la nica
combinacin lineal de ellos igual a cero, es aquella cuyos escalares son cero.
Es decir, si a1v1 + a2v2 + ... + anvn = 0 entonces a1 = a2 = a3 = ... = an = 0.
Definicin 3.7. Sean {v1, v2, ..., vn} n vectores de un espacio vectorial V.
Entonces se dice que los vectores son linealmente dependientes si existe una
combinacin lineal de ellos igual a cero, cuyos escalares no son todos cero.
Es decir existen a1, a2, a3,... ,an no todas cero tales que a1v1 + a2v2 + ... + anvn = 0.

Ejemplo 7
a) Consideremos los siguientes vectores en R4. v1 = (2, 1, 0, 3) y v2 = (6,
3, 0, 9).
Vamos a tomar una combinacin lineal de ellos igual a cero a1v1 + a2v2 = 0.
Entonces a1(2, 1, 0, 3) + a2(6, 3, 0, 9) = (0, 0, 0, 0), por lo tanto tenemos
2a1 6a2 = 0
el sistema a1 + 3a2 = 0 que si a1 = 3 y a2 = 1 se cumple la igualdad, por lo
3a1 9a2 = 0
tanto v1 y v2 son linealmente dependientes.
b) Consideremos los vectores en R3. v1 = (1, 2, 4) y v2 = (2, 5, 3).
Al tomar una combinacin lineal igual a cero b1v1 + b2v2 = 0 tenemos que
b1(1, 2, 4) + b2(2, 5, 3) = (0, 0, 0) de donde obtenemos el sistema

118

b1 + 2b2 = 0
2b1 + 5b2 = 0 cuya solucin es b1 = b2 = 0,
4b1 3b2 = 0

lgebralineal
por lo tanto v1 y v2 son linealmente independientes.
c) Determinar si los vectores de R3 v1 = (1, 3, 0) , v2 = (3, 0, 4) y v3 =
(11, 6, 12) son linealmente independientes o dependientes.
Consideremos una combinacin lineal de ellos igual a cero,
c1(1, 3, 0) + c2(3, 0, 4) + c3(11, 6, 12) = (0, 0, 0), entonces tenemos el
c1 + 3c2 + 11c3 = 0
c1 + 2c3 = 0
sistema de ecuaciones 3c1 6c3 = 0 de donde obtenemos que
c2 + 3c3 = 0
4c2 + 12c3 = 0
haciendo c3 = 1 obtenemos c2 = 3 y c1 = 2, por lo tanto v1, v2 y v3 son
linealmente dependientes.
Cuntos vectores deber tener un conjunto para ser linealmente
dependiente?
Teorema 3.6. Un conjunto de m vectores en Rn siempre es linealmente
dependiente si m > n.

Ejemplo 8
Consideremos el conjunto H = {(1,0,0), (0,1,0), (0,0,1), (1,2,3)} de 4 vectores
de R3 y una combinacin lineal de ellos igual a cero.
a (1,0,0) + b(0,1,0) + c(0,0,1) + d(1,2,3) = (0,0,0) entonces tenemos que:
a+d =0
a = d
b + 2d = 0 de donde obtenemos b = 2d el sistema tiene una infinidad de
c + 3d = 0
c = 3d
soluciones y por lo tanto el conjunto H es linealmente dependiente.
Corolario 3.1. Un conjunto de vectores linealmente independientes en Rn
contiene a lo ms n vectores.

Consideremos ahora un sistema homogneo (definicin 2.2.) de m ecuaciones


con n incgnitas.

119

Unidad 3
a11c1 + a12 c2 + ... + a1n cn = 0
a21c1 + a22 c2 + ... + a2 n cn = 0

am1c1 + am 2 c2 + + amn cn = 0
y sea la matriz asociada
a11

a
A= 21

am1

a12 ... a1n

a22 ... a2 n

am 2 ... amn

entonces tenemos el siguiente resultado.


Teorema 3.7. Las columnas de A, consideradas como vectores, son
linealmente dependientes si y slo si el sistema homogneo asociado tiene
soluciones diferentes de cero.

Ejemplo 9
x + 2 x2 x3 + 2 x4 = 0
Considera el sistema homogneo 1
y su matriz
3 x1 + 7 x2 + x3 + 4 x4 = 0
1 2 1 2 0
asociada A =
sus columnas son linealmente dependientes
1 4 0
3 7
(4 vectores en R2, teorema 3.6) por lo tanto, el sistema homogneo tiene ms
de una solucin no trivial. Vamos a encontrarla: Reduciendo por renglones
1 0 9
6 0
obtenemos

4 2 0
0 1
de donde el sistema asociado es x1 9 x3 + 6 x4 = 0 despejamos x1 y x2
x2 + 4 x3 2 x4 = 0
x1 = 9 x3 6 x4
x2 = 4 x3 + 2 x4
Se ve que este sistema tiene un nmero infinito de soluciones que se
pueden escribir como combinacin lineal de los vectores columna:

120

lgebralineal
x1 9 x3 6 x4
9

x2 = 4 x3 + 2 x4 = x 4
x3
3 1
x3

x4
0
x4

2
y
0

+ x 2
4 0

. Comprobaremos que

4
1

son soluciones linealmente independientes del sistema original.

Vamos a sustituir cada una de ellas en el sistema original:


9 + 2(4) 1 + 2(0) = 9 8 1 + 0 = 0
6 + 2(2) 0 + 2(1) = 6 + 4 + 2 = 0
3(9) + 7(4) + 1 + 4(0) = 27 28 + 1 + 0 = 0
3(6) + 7(2) + 0 + 4(1) = 18 + 14 + 4 = 0
por lo tanto (9, 4, 1, 0) y (6, 2, 0, 1) son soluciones del sistema original.
Probaremos ahora que son linealmente independientes:
Tomemos una combinacin lineal de ellos igual a cero:
a(9, 4, 1, 0) + b(6, 2, 0, 1) = 0
9a 6b = 0
a=0
4a + 2b = 0
entonces
de donde
b=0
a=0
b=0
por lo que (9, 4, 1, 0) y (6, 2, 0, 1) son linealmente independientes.
De aqu se desprende el siguiente teorema que agrupa varios resultados.

121

Unidad 3
Teorema 3.8. Sea A una matriz de nn. Entonces las siguientes afirmaciones
son equivalentes:
i)
ii)
iii)
iv)
v)
vi)

A es invertible.
La nica solucin al sistema homogneo Ax = 0 es la solucin trivial.
El sistema Ax = b tiene una solucin nica.
A es equivalente a la matriz identidad.
det A 0.
Las columnas de A (y sus renglones) son linealmente independientes.

Como consecuencia de los teoremas 3.5 y 3.6 tenemos el siguiente


resultado que nos ser muy til en la siguiente unidad.
Teorema 3.9. Cualquier conjunto de n vectores linealmente independientes
en Rn genera a Rn.
Como consecuencia de los teoremas 3.8 y 3.9 tres vectores en R3 generan a
R3, si y slo si, su determinante es diferente de cero.

Ejemplo 10
a) Los vectores (2, 1, 4), (1, 0, 2) y (3, 1, 5) generan a R3 ya que su
2 1
3
determinante 1
0 1 = 2(2) 1(5+4) +3(2) = 1 y por lo tanto son
4 2
5
linealmente independientes.
2
1 1 4
1 0
1 0 1
b) En M23 sean A1 =
y A3 =
, A2 =

2 3 0
3 1 1
1 2 1
Determinar si A1, A2 y A3 son linealmente independientes o dependientes.
Suponga que c1A1 + c2 A2 + c3A3 = 0,
1 0
entonces c1
3 1

122

2
1 1 4
1 0 1 0 0 0
+ c2
+ c3
=

1
2 3 0
1 2 1 0 0 0

S-ar putea să vă placă și